Walk one round

A 4 digit number is randomly picked out of all the 4 digit number. Find the probability that the product of its digits is divisible by 3. If the answer can be expressed as a b \large\frac{a}{b} for coprime positive integers a , b a,b , what is the value of a + b a+b ? This problem is part of the set Advanced is basic

Note: 012 is a two digit number, not a three digit number.


The answer is 232.

This section requires Javascript.
You are seeing this because something didn't load right. We suggest you, (a) try refreshing the page, (b) enabling javascript if it is disabled on your browser and, finally, (c) loading the non-javascript version of this page . We're sorry about the hassle.

1 solution

It is easier to calculate the complement first. For a 4 4 -digit number to not have the product of its digits be divisible by 3 , 3, none of the digits can be divisible by 3. 3. Now there are 6 6 digits not divisible by 3 , 3, (namely 1 , 2 , 4 , 5 , 7 , 8 1,2,4,5,7,8 ), and the number of 4 4 -digit numbers that can be formed using just these digits is 6 4 = 1296. 6^{4} = 1296.

Since there are 9000 9000 4 4 -digit numbers in all, the probability the the digits of a randomly chosen 4 4 -digit numbers has a "digit product" divisible by 3 3 is

1 1296 9000 = 107 125 , 1 - \dfrac{1296}{9000} = \dfrac{107}{125}, and thus a + b = 107 + 125 = 232 . a + b = 107 + 125 = \boxed{232}.

Moderator note:

Yes, probability by complement is the easiest way for this question.

Yep. that's right . :)

Keshav Tiwari - 6 years, 2 months ago

Log in to reply

Damn,made a silly mistake,took 107+125 as 222!!:P

Adarsh Kumar - 6 years, 2 months ago

Log in to reply

Well you still had another try .:P

Keshav Tiwari - 6 years, 2 months ago

Log in to reply

@Keshav Tiwari Long story.

Adarsh Kumar - 6 years, 2 months ago

Aw... I put 143. I thought the probability was 18 125 \frac { 18 }{ 125 } . I forgot to subtract it from 1.

Oliver Daugherty-Long - 6 years, 1 month ago

Log in to reply

I did the same thing on my first attempt; I only got the correct answer on my second attempt. :)

Brian Charlesworth - 6 years, 1 month ago

0 pending reports

×

Problem Loading...

Note Loading...

Set Loading...